2
$\begingroup$

Let $X$ be a smooth irreducible variety over $\mathbb{C}$ and $Y$ be a smooth algebraic space proper over $\mathbb{C}$. Assume $f:X \to Y$ is a morphism such that bijective on $\mathbb{C}$-points.

Question: Is $f$ an isomorphism? Hence $Y$ is a variety?

When $Y$ is already a variety, this is true from Zariski's Main Theorem (see for example Bijection implies isomorphism for algebraic varieties).

$\endgroup$
5
  • 2
    $\begingroup$ $Y$ can be considered a compact complex manifold by Artin and then $f$ is a holomorphic bijection. Holomorphic bijections have holomorphic inverses. $\endgroup$
    – user178279
    May 29, 2021 at 9:15
  • $\begingroup$ @virkkunen Thanks! Is there any reference about this fact? I mean consider $Y$ as a complex manifold? $\endgroup$
    – Kim
    May 29, 2021 at 10:02
  • 1
    $\begingroup$ By miracle flatness (Matsumura, Comm. Ring Theory, Thm 23.1) the morphism $f$ is flat, hence open, hence a homeomorphism, so $Y$ is integral. $\endgroup$ May 29, 2021 at 10:51
  • 5
    $\begingroup$ The same proof from the answer that you link works for algebraic spaces as well as for schemes: Zariski's Main Theorem works for algebraic spaces. $\endgroup$ May 29, 2021 at 12:34
  • 2
    $\begingroup$ @MatthieuRomagny the question is not whether $Y$ is integral (as opposed to just of finite type), but whether $Y$ is even a scheme. $\endgroup$ May 29, 2021 at 16:30

1 Answer 1

1
$\begingroup$

Edit: Not an answer to the question. I confused the roles of $X$ and $Y$.

If $X\to Y$ is a quasi-finite separated morphism of algebraic spaces and $Y$ is a scheme, then $X$ is a scheme.

This is, as Jason Starr already commented, a consequence of Zariski's Main Theorem for algebraic spaces.

You are thus reduced to the case of varieties.

$\endgroup$
2
  • 2
    $\begingroup$ I believe the more relevant result has hypothesis that $X$ is a scheme, rather than that $Y$ is a scheme, cf. mathoverflow.net/questions/4573/… $\endgroup$ May 30, 2021 at 23:11
  • $\begingroup$ @JasonStarr Whoops, you are right. I misread the roles of $X$ and $Y$. $\endgroup$ Jun 6, 2021 at 16:32

Your Answer

By clicking “Post Your Answer”, you agree to our terms of service and acknowledge you have read our privacy policy.

Not the answer you're looking for? Browse other questions tagged or ask your own question.